Diễn Đàn MathScopeDiễn Đàn MathScope
  Diễn Đàn MathScope
Ghi Danh Hỏi/Ðáp Thành Viên Social Groups Lịch Ðánh Dấu Ðã Ðọc

Go Back   Diễn Đàn MathScope > Sơ Cấp > Việt Nam và IMO > 2011

News & Announcements

Ngoài một số quy định đã được nêu trong phần Quy định của Ghi Danh , mọi người tranh thủ bỏ ra 5 phút để đọc thêm một số Quy định sau để khỏi bị treo nick ở MathScope nhé !

* Nội quy MathScope.Org

* Một số quy định chung !

* Quy định về việc viết bài trong diễn đàn MathScope

* Nếu bạn muốn gia nhập đội ngũ BQT thì vui lòng tham gia tại đây

* Những câu hỏi thường gặp

* Về việc viết bài trong Box Đại học và Sau đại học


Trả lời Gởi Ðề Tài Mới
 
Ðiều Chỉnh Xếp Bài
Old 19-07-2011, 11:17 PM   #31
kien10a1
+Thành Viên+
 
kien10a1's Avatar
 
Tham gia ngày: Feb 2011
Đến từ: Vĩnh Yên- Vĩnh Phúc
Bài gởi: 371
Thanks: 43
Thanked 263 Times in 153 Posts
Gửi tin nhắn qua Yahoo chát tới kien10a1
Trích:
Nguyên văn bởi Traum View Post
Bài 4:

Gọi $S(n) $ là số cách cân thỏa mãn.
Xét với $n+1 $ quả cân.

Nếu lần thứ $n+1 $, quả cân nặng nhất được đặt lên thì nó sẽ phải nằm bên trái. Do đó ta có $S(n-1) $ cách đặt thỏa mãn.

Nếu lần thứ $n+1 $, quả cân $2^k $ nào đó được đặt lên thì ta có với quả cân này có $2 $ lựa chọn để đặt, phải trái tùy ý, do quả cân $2^n $ đã nằm ở bên trái rồi. Do có tất cả là $n $ cách chọn $k $, do đó ta có tất cả $2nS(n) $ cách đặt thỏa mãn.

Vậy tổng cộng ta có $S(n+1) = 2nS(n) + S(n) = (2n+1)S(n) $.
Kết hợp với $S(1) = 1 $, ta thu được $S(n) = 1\times 3\times...\times (2n-1) $.
Em nghĩ là anh vẫn nên có nhận xét trên của em mới khẳng định được đặt n quả cân khối lượng bất kì dạng $2^k $ là $S(n) $
[RIGHT][I][B]Nguồn: MathScope.ORG[/B][/I][/RIGHT]
 
__________________
Quay về với nơi bắt đầu
kien10a1 is offline   Trả Lời Với Trích Dẫn
Old 19-07-2011, 11:28 PM   #32
huynhcongbang
Administrator

 
huynhcongbang's Avatar
 
Tham gia ngày: Feb 2009
Đến từ: Ho Chi Minh City
Bài gởi: 2,413
Thanks: 2,165
Thanked 4,188 Times in 1,381 Posts
Gửi tin nhắn qua Yahoo chát tới huynhcongbang
Dưới đây là lời giải của một bạn bên mathlink, mọi người tham khảo thử! Xét trường hợp như hình vẽ dưới đây, các trường hợp khác chứng minh tương tự.



Gọi tam giác tạo thành bởi các đường thẳng $\l_a, \l_b, \l_c $ tương ứng là $A_1,B_1,C_1 $. Gọi P là tiếp điểm của đường thẳng $\l $ với $\Gamma $ và P' là điểm đối xứng với P qua BC. Dễ thấy P' thuộc $B_1C_1 $.

Trước hết, ta sẽ chứng minh rằng các đường thẳng $A_1A, B_1B, C_1C $ đồng quy tại một điểm nằm trên $\Gamma $.
Thật vậy, do $A_1C_1 $ đối xứng với $\l $ qua AC nên A cách đều hai đường thẳng $\l, A_1C_1 $, tuy nhiên do A cũng cách đều hai đường thẳng $\l, A_1B_1 $ nên A cách đều hai đường thẳng $A_1B_1, A_1C_1 $ hay $AA_1 $ là phân giác góc $\widehat{B_1A_1C_1} $.
Tương tự, ta cũng có $BB_1, CC_1 $ là các phân giác của tam giác $A_1B_1C_1 $. Suy ra các đường thẳng $AA_1, BB_1, CC_1 $ đồng quy, giả sử điểm đồng quy là I. Ta cần chứng minh I thuộc $\Gamma $.
Ta tính được $\widehat{B_1A_1C_1} = 180^0- 2\widehat{BAC} $ nên $\widehat{BIC} = \frac{1}{2}(\widehat{B_1} + \widehat{C_1}) = \widahat{BAC} $, tức là 4 điểm A, B, C, I cùng thuộc 1 đường tròn.

Gọi Q là giao điểm của $(BB_1P'), (CC_1P') $. Ta có:
$\widehat{BQP'} + \widehat{CQP'} = \widehat{BB_1P'}+\widehat{CC_1P'} = \widehat{BAC} $ nên bốn điểm A, B, C, Q cùng thuộc 1 đường tròn.
Ta cũng có:
$\widehat{B_1QC_1}= \widehat{B_1QB}+ \widehat{BQC}+ \widehat{CQC_1} = \widehat{B_1P'B}+\widehat{BAC}+\widehat{C_1P'C} = \widehat{BAC} + 180^0-\widehat{BPC}=2 \widehat{BAC} $. Suy ra bốn điểm $A_1, B_1, C_1, Q $ cùng thuộc 1 đường tròn.
Do đó, các đường tròn $\Gamma, (A_1B_1C_1) $ cắt nhau tại Q.
Cũng bằng biến đổi góc, ta thấy tiếp tuyến tại Q của $(\Gamma) $ trùng với tiếp tuyến tại Q của $(A_1B_1C_1) $ nên hai đường tròn này tiếp xúc với nhau tại Q. Ta có đpcm.


[RIGHT][I][B]Nguồn: MathScope.ORG[/B][/I][/RIGHT]
 

thay đổi nội dung bởi: huynhcongbang, 19-07-2011 lúc 11:50 PM Lý do: LATEX
huynhcongbang is offline   Trả Lời Với Trích Dẫn
The Following 3 Users Say Thank You to huynhcongbang For This Useful Post:
kimlinh (20-07-2011), Lan Phuog (20-07-2011), n.v.thanh (20-07-2011)
Old 19-07-2011, 11:34 PM   #33
Traum
Moderator
 
Traum's Avatar
 
Tham gia ngày: Nov 2007
Đến từ: cyber world
Bài gởi: 413
Thanks: 14
Thanked 466 Times in 171 Posts
Trích:
Nguyên văn bởi kien10a1 View Post
Em nghĩ là anh vẫn nên có nhận xét trên của em mới khẳng định được đặt n quả cân khối lượng bất kì dạng $2^k $ là $S(n) $
À tất nhiên, cảm ơn em.
[RIGHT][I][B]Nguồn: MathScope.ORG[/B][/I][/RIGHT]
 
__________________
Traum is giấc mơ.
Traum is offline   Trả Lời Với Trích Dẫn
Old 19-07-2011, 11:52 PM   #34
kien10a1
+Thành Viên+
 
kien10a1's Avatar
 
Tham gia ngày: Feb 2011
Đến từ: Vĩnh Yên- Vĩnh Phúc
Bài gởi: 371
Thanks: 43
Thanked 263 Times in 153 Posts
Gửi tin nhắn qua Yahoo chát tới kien10a1
Trích:
Nguyên văn bởi huynhcongbang View Post
Dưới đây là lời giải của một bạn bên mathlink, mọi người tham khảo thử! Xét trường hợp như hình vẽ dưới đây, các trường hợp khác chứng minh tương tự.



Gọi tam giác tạo thành bởi các đường thẳng $\l_a, \l_b, \l_c $ tương ứng là $A_1,B_1,C_1 $. Gọi P là tiếp điểm của đường thẳng $\l $ với $\Gamma $ và P' là điểm đối xứng với P qua BC. Dễ thấy P' thuộc $B_1C_1 $.

Trước hết, ta sẽ chứng minh rằng các đường thẳng $A_1A, B_1B, C_1C $ đồng quy tại một điểm nằm trên $\Gamma $.
Thật vậy, do $A_1C_1 $ đối xứng với $\l $ qua AC nên A cách đều hai đường thẳng $\l, A_1C_1 $, tuy nhiên do A cũng cách đều hai đường thẳng $\l, A_1B_1 $ nên A cách đều hai đường thẳng $A_1B_1, A_1C_1 $ hay $AA_1 $ là phân giác góc $\widehat{B_1A_1C_1} $.
Tương tự, ta cũng có $BB_1, CC_1 $ là các phân giác của tam giác $A_1B_1C_1 $. Suy ra các đường thẳng $AA_1, BB_1, CC_1 $ đồng quy, giả sử điểm đồng quy là I. Ta cần chứng minh I thuộc $\Gamma $.
Ta tính được $\widehat{B_1A_1C_1} = 180^0- 2\widehat{BAC} $ nên $\widehat{BIC} = \frac{1}{2}(\widehat{B_1} + \widehat{C_1}) = \widahat{BAC} $, tức là 4 điểm A, B, C, I cùng thuộc 1 đường tròn.

Gọi Q là giao điểm của $(BB_1P'), (CC_1P') $. Ta có:
$\widehat{BQP'} + \widehat{CQP'} = \widehat{BB_1P'}+\widehat{CC_1P'} = \widehat{BAC} $ nên bốn điểm A, B, C, Q cùng thuộc 1 đường tròn.
Ta cũng có:
$\widehat{B_1QC_1}= \widehat{B_1QB}+ \widehat{BQC}+ \widehat{CQC_1} = \widehat{B_1P'B}+\widehat{BAC}+\widehat{C_1P'C} = \widehat{BAC} + 180^0-\widehat{BPC}=2 \widehat{BAC} $. Suy ra bốn điểm $A_1, B_1, C_1, Q $ cùng thuộc 1 đường tròn.
Do đó, các đường tròn $\Gamma, (A_1B_1C_1) $ cắt nhau tại Q.
Cũng bằng biến đổi góc, ta thấy tiếp tuyến tại Q của $(\Gamma) $ trùng với tiếp tuyến tại Q của $(A_1B_1C_1) $ nên hai đường tròn này tiếp xúc với nhau tại Q. Ta có đpcm.

Em làm đến chỗ A,B,C,I thì chịu. Nhưng cái chỗ cuối biến đổi góc có chắc không ạ?
[RIGHT][I][B]Nguồn: MathScope.ORG[/B][/I][/RIGHT]
 
__________________
Quay về với nơi bắt đầu
kien10a1 is offline   Trả Lời Với Trích Dẫn
Old 20-07-2011, 12:37 AM   #35
tuan119
+Thành Viên+
 
tuan119's Avatar
 
Tham gia ngày: Dec 2008
Bài gởi: 993
Thanks: 273
Thanked 666 Times in 422 Posts
Các bạn có thể tham khảo thêm tại đây:
Câu 4:
[Only registered and activated users can see links. ]

Câu 5:
[Only registered and activated users can see links. ]

Câu 6:
[Only registered and activated users can see links. ]
[RIGHT][I][B]Nguồn: MathScope.ORG[/B][/I][/RIGHT]
 
__________________
$\bf{T}\mathcal{smile} $__________________________________________________ ________________
tuan119 is offline   Trả Lời Với Trích Dẫn
The Following 2 Users Say Thank You to tuan119 For This Useful Post:
kimlinh (20-07-2011), n.v.thanh (20-07-2011)
Old 20-07-2011, 01:19 AM   #36
cleverboy
+Thành Viên+
 
Tham gia ngày: Dec 2008
Bài gởi: 108
Thanks: 17
Thanked 58 Times in 32 Posts
Lời giải bài 6 IMO 2011!
[RIGHT][I][B]Nguồn: MathScope.ORG[/B][/I][/RIGHT]
 
File Kèm Theo
Kiểu File : doc Bài VI IMO 2011.doc (87.5 KB, 145 lần tải)

thay đổi nội dung bởi: cleverboy, 20-07-2011 lúc 08:57 AM
cleverboy is offline   Trả Lời Với Trích Dẫn
The Following 2 Users Say Thank You to cleverboy For This Useful Post:
kimlinh (20-07-2011), lexuanthang (20-07-2011)
Old 20-07-2011, 06:42 AM   #37
Lan Phuog
+Thành Viên Danh Dự+
 
Lan Phuog's Avatar
 
Tham gia ngày: Mar 2010
Đến từ: Thái Bình
Bài gởi: 564
Thanks: 289
Thanked 326 Times in 182 Posts
Trích:
Nguyên văn bởi hizact View Post
NGÀY 2

Bài 4. Giả sử $n > 0 $ là một số nguyên. Cho một cái cân hai đĩa và $n $ quả cân với trọng lượng là ${2^0},{2^1},...,{2^{n - 1}} $. Ta muốn đặt lên cái cân mỗi một trong $n $ quả cân, lần lượt từng quả một, theo cách để bảo đảm đĩa cân bên phải không bao giờ nặng hơn đĩa cân bên trái. Ở mỗi bước ta chọn một trong các quả cân chưa được đặt lên cân, rồi đặt nó hoặc vào đĩa bên trái, hoặc vào đĩa bên phải, cho đến khi tất cả các quả cân đều đã được đặt lên cân. Xác định xem có bao nhiêu cách để thực hiện được mục đích đề ra
Bài này mình giải theo hướng này:
Gọi $S_n $ là số cách đặt ứng với n quả cân.
Xét $n+1 $ quả $2^0,2^1,...,2^{n-1},2^n $
i,nếu quả $2^n $ được chọn để đặt cuối cùng: sẽ có $S_n $ cách.
ii,nếu quả $2^n $ được chọn ở bước thứ $k=1,2,...,n $. Tất nhiên quả này phải đặt bên trái vì $2^n>2^{n-1}+2^{n-2}+...+2^1+2^0 $
Có n cách chọn. còn lại n quả, sẽ có $2nS_n $ cách
Như vậy $S_{n+1}=(2n+1)S_n $
------------------------------
Trích:
Nguyên văn bởi huynhcongbang View Post
Dưới đây là lời giải của một bạn bên mathlink, mọi người tham khảo thử! Xét trường hợp như hình vẽ dưới đây, các trường hợp khác chứng minh tương tự.



Gọi tam giác tạo thành bởi các đường thẳng $\l_a, \l_b, \l_c $ tương ứng là $A_1,B_1,C_1 $. Gọi P là tiếp điểm của đường thẳng $\l $ với $\Gamma $ và P' là điểm đối xứng với P qua BC. Dễ thấy P' thuộc $B_1C_1 $.

Trước hết, ta sẽ chứng minh rằng các đường thẳng $A_1A, B_1B, C_1C $ đồng quy tại một điểm nằm trên $\Gamma $.
Thật vậy, do $A_1C_1 $ đối xứng với $\l $ qua AC nên A cách đều hai đường thẳng $\l, A_1C_1 $, tuy nhiên do A cũng cách đều hai đường thẳng $\l, A_1B_1 $ nên A cách đều hai đường thẳng $A_1B_1, A_1C_1 $ hay $AA_1 $ là phân giác góc $\widehat{B_1A_1C_1} $.
Tương tự, ta cũng có $BB_1, CC_1 $ là các phân giác của tam giác $A_1B_1C_1 $. Suy ra các đường thẳng $AA_1, BB_1, CC_1 $ đồng quy, giả sử điểm đồng quy là I. Ta cần chứng minh I thuộc $\Gamma $.
Ta tính được $\widehat{B_1A_1C_1} = 180^0- 2\widehat{BAC} $ nên $\widehat{BIC} = \frac{1}{2}(\widehat{B_1} + \widehat{C_1}) = \widahat{BAC} $, tức là 4 điểm A, B, C, I cùng thuộc 1 đường tròn.

Gọi Q là giao điểm của $(BB_1P'), (CC_1P') $. Ta có:
$\widehat{BQP'} + \widehat{CQP'} = \widehat{BB_1P'}+\widehat{CC_1P'} = \widehat{BAC} $ nên bốn điểm A, B, C, Q cùng thuộc 1 đường tròn.
Ta cũng có:
$\widehat{B_1QC_1}= \widehat{B_1QB}+ \widehat{BQC}+ \widehat{CQC_1} = \widehat{B_1P'B}+\widehat{BAC}+\widehat{C_1P'C} = \widehat{BAC} + 180^0-\widehat{BPC}=2 \widehat{BAC} $. Suy ra bốn điểm $A_1, B_1, C_1, Q $ cùng thuộc 1 đường tròn.
Do đó, các đường tròn $\Gamma, (A_1B_1C_1) $ cắt nhau tại Q.
Cũng bằng biến đổi góc, ta thấy tiếp tuyến tại Q của $(\Gamma) $ trùng với tiếp tuyến tại Q của $(A_1B_1C_1) $ nên hai đường tròn này tiếp xúc với nhau tại Q. Ta có đpcm.

Làm đoạn tâm nội tiếp A'B'C' nằm trên đường tròn ngoại tiếp ABC là chịu,không có phương hướng.
[RIGHT][I][B]Nguồn: MathScope.ORG[/B][/I][/RIGHT]
 

thay đổi nội dung bởi: Lan Phuog, 20-07-2011 lúc 06:53 AM Lý do: Tự động gộp bài
Lan Phuog is offline   Trả Lời Với Trích Dẫn
Old 20-07-2011, 07:22 AM   #38
pabopit
+Thành Viên+
 
Tham gia ngày: Jan 2011
Bài gởi: 77
Thanks: 29
Thanked 58 Times in 41 Posts
Em có 1 lời giải cho bài 3.Không biết có nhầm chỗ nào không.

$f(x+y)\leq yf(x)+f(f(x)) $ (0)
Đặt $f(0)=a $
Ký hiệu $P(x,y) $ là các bộ (x,y) thay thế vào (0)
1)Xét $P(x,0)\Rightarrow f(x)\leq f(f(x)) $
2)Xét $P(0,y)\Rightarrow f(y)\leq ya+f(a) $
3)Xét $P(x,f(x)-x)\Rightarrow (f(x)-x)f(x)\geq 0 $
4)Xét $P(x,-x) \Rightarrow a\leq f(f(x))-xf(x) $
Bước 1: Ta chứng minh $f(x)\leq f(a) $ với mọi $x $
Thật vậy Từ 0) và 2) thì $f(x+y)\leq yf(x)+af(x)+f(a) $ (5)
Cho $y=-a $ thì $f(x)\leq f(a) $ với mọi x
Bước 2:Từ 4) và bước 1 suy ra $xf(x)\leq f(f(x))-a\leq f(a)-a $
B3: Xét $y<0 $ và từ 2) suy ra $yf(y)\geq y^2.a+yf(a) $
Kết hợp B2 suy ra $y^2.a+yf(a)\leq f(a)-a $ với mọi y<0.
Suy ra $a\leq 0 $
B4:Nếu a=0 thì $f(a)=0 $
Từ 2) suy ra $f(y)\leq 0 $ với mọi y
Từ B2 suy ra $xf(x)\leq 0 $ với mọi x.
Xét $x<0 $.Nếu $f(x)<0 $ thì $ xf(x)>0 $,vô lý

B5: Nếu a<0.Từ 5) cho $x=a,y=0 $ thì $f(a)\leq af(a)+f(a) $
$\Rightarrow af(a)\geq 0 $.Mà $a<0 $ suy ra $f(a)\leq 0 $
Nếu $f(a)=0 $ thì từ 1) cho $x=a \Rightarrow 0\leq f(f(a))=f(0) $,vô lý
$\Rightarrow f(a)< 0 $
Từ 3) cho $x=a\Rightarrow f(a)<a=f(0) $,trái với b1.
[RIGHT][I][B]Nguồn: MathScope.ORG[/B][/I][/RIGHT]
 

thay đổi nội dung bởi: pabopit, 20-07-2011 lúc 07:32 AM
pabopit is offline   Trả Lời Với Trích Dẫn
Old 20-07-2011, 10:10 AM   #39
phamtoan
Banned
 
Tham gia ngày: Apr 2011
Đến từ: VMF
Bài gởi: 313
Thanks: 266
Thanked 63 Times in 50 Posts
Gửi tin nhắn qua Yahoo chát tới phamtoan
IMO năm nay mọi người Cảm thấy thế nào!
[RIGHT][I][B]Nguồn: MathScope.ORG[/B][/I][/RIGHT]
 
phamtoan is offline   Trả Lời Với Trích Dẫn
Old 20-07-2011, 11:43 AM   #40
Lan Phuog
+Thành Viên Danh Dự+
 
Lan Phuog's Avatar
 
Tham gia ngày: Mar 2010
Đến từ: Thái Bình
Bài gởi: 564
Thanks: 289
Thanked 326 Times in 182 Posts
Với ý tưởng như bài 6, mình xin đề xuất bài toán sau:

Cho tam giác $A'B'C' $ nội tiếp đường tròn tâm $O' $ và ngoại tiếp đường tròn tâm $I $. Một đường tròn tâm $O $ đi qua I cắt $IA,IB,IC $ lần lượt tại $A,B,C $. $BC,CA,AB $ tương ứng cắt $B'C',C'A',A'B' $ tại $D,E,F $. Chứng minh: 3 điểm $D,E,F $ thẳng hàng và đường thẳng đi qua nó tiếp xúc với $(O) $ khi và chỉ khi $(O) $ tiếp xúc trong với $(O') $
[RIGHT][I][B]Nguồn: MathScope.ORG[/B][/I][/RIGHT]
 

thay đổi nội dung bởi: Lan Phuog, 20-07-2011 lúc 11:44 AM Lý do: Tự động gộp bài
Lan Phuog is offline   Trả Lời Với Trích Dẫn
Old 20-07-2011, 01:54 PM   #41
cleverboy
+Thành Viên+
 
Tham gia ngày: Dec 2008
Bài gởi: 108
Thanks: 17
Thanked 58 Times in 32 Posts
Bài 6 còn có thể phát biểu dưới dạng sau:
Cho tứ giác $ABCD $. Gọi $E $ là giao của $AB $ và $CD $;$F $ là giao điểm của $AD $ và $BC $. Các đường phân giác trong của các góc đỉnh $C $, $D $, $E $ của tứ giác toàn phần $ABCDEF $ cắt nhau tạo thành tam giác $MNP $. Gọi $(C) $ là đường tròn ngoại tiếp tam giác $MNP $.Chứng minh rằng $(C) $ tiếp xúc với đường tròn ngoại tiếp tam giác $ABF $ khi và chỉ khi $(C) $ tiếp xúc với $CD $.
[RIGHT][I][B]Nguồn: MathScope.ORG[/B][/I][/RIGHT]
 

thay đổi nội dung bởi: cleverboy, 20-07-2011 lúc 01:57 PM
cleverboy is offline   Trả Lời Với Trích Dẫn
Old 21-07-2011, 09:14 PM   #42
phuanxu
+Thành Viên+
 
Tham gia ngày: Jul 2011
Bài gởi: 3
Thanks: 0
Thanked 3 Times in 3 Posts
Mình mới tham gia diễn đàn. xin có chút nhận xét về đề IMO năm nay. so với mấy năm thì đề IMO năm nay cũng như vậy, có điều bài 3 và bài 5 không khó như mấy năm. bài 6 là khó nhất. bài 1 và 4 tương đối dể, đặc biệt bài 4 là bài tổ hợp nhưng khá đơn giản. bài 1 là bài số học cơ bản. bài 2 không khó lắm nhưng cũng không tầm thường. nói chung đề năm nay khá hay. dự đoán đoàn việt nam 2:2:2
[RIGHT][I][B]Nguồn: MathScope.ORG[/B][/I][/RIGHT]
 
phuanxu is offline   Trả Lời Với Trích Dẫn
Old 22-07-2011, 09:31 AM   #43
phuanxu
+Thành Viên+
 
Tham gia ngày: Jul 2011
Bài gởi: 3
Thanks: 0
Thanked 3 Times in 3 Posts
Xin đóng góp 1 lời giải dễ hiểu và ngắn gọn cho bài 6 (không biết vẽ hình trên này mong các bạn thông cảm)
Gọi $ A_1, B_1, C_1 $ lần lượt là giao điểm của các cặp đường thẳng $(l_b, l_c); (l_a, l_c); (l_a, l_b). $
Gọi M là tiếp điểm của l với đường tròn (T) và B’, C’ lần lượt là các điểm đối xứng của M qua AC, AB.
1) dễ thấy tứ giác $AB'A_1C $ nội tiếp đường tròn
2) $ AA_1, BB_1, CC_1 $ đồng quy tại D
Đường tròn$ (AB'A_1C) $ cắt đường tròn (T) tại I.
Sử dụng (1) và (2) và để ý các góc nội tiếp bằng nhau dễ thấy tứ giác $BIB_1C' $nội tiếp
Từ đó suy ra IB là tia phân giác của góc $MIB_1 $
Suy ra AC’,$ IB_1 $ và (T) đồng quy tại E
Tương tự CB’, $IA_1 $ và (T) đồng quy tại F
Suy ra EF song song với $A_1B_1 $ hay (T) tiếp xúc với $(A_1B_1C_1) $ tại I.


Moderator note: học gõ Latex nha bạn, nếu không lần sau sẽ bị xóa bài.
[RIGHT][I][B]Nguồn: MathScope.ORG[/B][/I][/RIGHT]
 

thay đổi nội dung bởi: sang89, 22-07-2011 lúc 10:15 AM
phuanxu is offline   Trả Lời Với Trích Dẫn
Old 26-07-2011, 09:00 AM   #44
nguyenlevan
+Thành Viên+
 
Tham gia ngày: Jul 2011
Bài gởi: 2
Thanks: 0
Thanked 1 Time in 1 Post
Lời giải bài 4 IMO
Vì quả cân có khối lượng $2^{n-1} $ nặng hơn tổng khối lượng của n-1 quả còn lại nên bất kỳ một cách đặt lần lượt từng quả một lên đĩa thỏa mãn “đĩa cân bên phải không bao giờ nặng hơn đĩa cân bên trái”(*) bắt buộc quả có khối lượng $2^{n-1} $ phải được đặt ở đĩa bên trái.
Gọi $A_n $ là số cách đặt n quả cân lên đĩa thỏa mãn (*),$A_k $ là số cách đặt k quả cân có khối lượng $2^0,2^1,...,2^{k-1} $ (dễ thấy $A_k $ cũng là số cách đặt k quả cân bất kỳ trong số n quả vào đĩa thỏa mãn (*)), $B_k $là số cách đặt n quả cân lên đĩa thỏa mãn (*) và lần đặt thứ k đặt quả có khối lượng vào đĩa bên trái thì $A_n=\sum_{k= 1}^nB_k $ .
Để tính $A_n $ ta tính $B_k $ . Có $C_{n-1}^{k-1} $ cách chọn ra k-1 quả cân trong số n-1 quả còn lại, với mỗi cách chọn đó lại có $A_{k-1} $ cách để lần lượt nó lên đĩa thỏa mãn (*), với mỗi cách này lại có $(n-k)!2^{n-k} $ cách đặt nốt n-k quả còn lại lên đĩa (Vì nếu đã xếp quả có khối lượng ở bước k thì n-k bước còn lại có thể xếp tùy ý vào bên phải, trái mà vẫn thỏa mãn (*)). Theo quy tắc nhân ta có $B_k= C_{n-1}^{k-1}(n-k)!2^{n-k}^A_{k-1} $ .
Do vậy $A_n= \sum_{k=1}^nC_{n-1}^{k-1}(n-k)!2^{n-k}A_{k-1} $ (với $A_0= A_1= 1 $ ) là kết quả phải tìm
[RIGHT][I][B]Nguồn: MathScope.ORG[/B][/I][/RIGHT]
 

thay đổi nội dung bởi: nguyenlevan, 26-07-2011 lúc 04:20 PM Lý do: đã sửa trực tiếp
nguyenlevan is offline   Trả Lời Với Trích Dẫn
The Following User Says Thank You to nguyenlevan For This Useful Post:
huynhcongbang (07-08-2011)
Old 05-08-2011, 01:35 PM   #45
n.v.thanh
Moderator
 
n.v.thanh's Avatar
 
Tham gia ngày: Nov 2009
Bài gởi: 2,849
Thanks: 2,980
Thanked 2,537 Times in 1,008 Posts
Đáp án chính thức của kì thi IMO 2011.Được cắt ra từ IMO Shortlist 2011:
[Only registered and activated users can see links. ]
[Only registered and activated users can see links. ]
Nguồn: Daji

[RIGHT][I][B]Nguồn: MathScope.ORG[/B][/I][/RIGHT]
 
File Kèm Theo
Kiểu File : pdf 2011_imo_final6.pdf (150.1 KB, 358 lần tải)

thay đổi nội dung bởi: n.v.thanh, 05-08-2011 lúc 02:25 PM
n.v.thanh is offline   Trả Lời Với Trích Dẫn
The Following 5 Users Say Thank You to n.v.thanh For This Useful Post:
chemmath (10-08-2011), perfectstrong (07-11-2011), thaipanh8 (13-08-2011), The Swastika (06-08-2011), tranghieu95 (08-10-2011)
Trả lời Gởi Ðề Tài Mới

Bookmarks

Ðiều Chỉnh
Xếp Bài

Quuyền Hạn Của Bạn
You may not post new threads
You may not post replies
You may not post attachments
You may not edit your posts

BB code is Mở
Smilies đang Mở
[IMG] đang Mở
HTML đang Tắt

Chuyển đến


Múi giờ GMT. Hiện tại là 03:53 PM.


Powered by: vBulletin Copyright ©2000-2024, Jelsoft Enterprises Ltd.
Inactive Reminders By mathscope.org
[page compression: 114.14 k/130.58 k (12.59%)]